গরম জলের ট্যাঙ্ক থেকে তাপ স্থানান্তর গণনা করার জন্য আপনি কীভাবে একটি অন্তরণ জ্যাকেটের আর-মান ব্যবহার করবেন?


0

আমি জানি কার্টেসিয়ান এবং নলাকার স্থানাঙ্কগুলিতে তাপ স্থানান্তর গণনা করতে কীভাবে তাপ প্রতিরোধের সার্কিটগুলি ব্যবহার করতে হয়, তবে বাস্তব-বিশ্বের অন্তরণ মূল্যগুলি কীভাবে মোকাবেলা করতে হয় তা আমি জানি না এবং আমি এই অনলাইন সম্পর্কে খুব বেশি কিছু খুঁজে পেতে সক্ষম হইনি।

উদাহরণস্বরূপ, আমার কাছে এই গরম জলের ট্যাঙ্ক রয়েছে যার সাথে 2 "আর -12.5 ইনসুলেশন রয়েছে একটি ইনসুলেশন জ্যাকেট রয়েছে simp সরলতার জন্য, আসুন নিরোধকটি একমাত্র জিনিস যা শীতল বাইরের বায়ু থেকে গরম জলকে আলাদা করে দেয় (ট্যাঙ্কের নীচ থেকে কোনও তাপ স্থানান্তর নয়) , কেবলমাত্র অন্তরণ জ্যাকেটের মাধ্যমে ট্যাঙ্কের শীর্ষস্থানীয়) water 2 "ইনসুলেশন জ্যাকেটটি নগণ্য নয়, জলের ট্যাঙ্কের বেধকে ধরে রাখুন।

সুতরাং এটি গরম জল যায় -> নিরোধক -> বাইরে ঠান্ডা বাতাস। আমি ট্যাঙ্কের গরম জলের জন্য একটি নিখরচায় পরিবাহী সহগ খুঁজে পেতে পারি এবং বায়ুর জন্য একটি সংবাহন সহগও সন্ধান করতে পারি। এটা আমার সূত্র ব্যবহার করার অনুমতি দেয় (একবার দুইবার জল এবং একবার বায়ু জন্য)।

R=1/(h2pirheightofinsulationjacket)

সুতরাং সমস্যাটি কীভাবে ইনসুলেশন জ্যাকেটটি মোকাবেলা করতে হয়। আমি কীভাবে 2 "আর -12.5 অন্তরণটি ব্যাখ্যা করব? আমি কীভাবে নিরোধকের ঘনত্বকে ফ্যাক্টর করব? তাপ স্থানান্তর সমীকরণে পৃষ্ঠের ক্ষেত্রটি যত্ন নেওয়া হয়।

Q=(TfTi)/R
Q=(TfTi)/RA

আপডেট হয়েছে: নলাকার স্থানাঙ্কগুলিতে আমি কীভাবে একটি আর-মান ব্যবহার করব?

উদাহরণস্বরূপ

R/A=1/(hW2pir1)+Rins/(whatAreaHere)+1/(hA2pir2)

আমি আমার আর-মান অন্তরণ জন্য অঞ্চলটি মিস করছি

উত্তর:


2

নিরোধক আর-মানটি ইতিমধ্যে বেধের জন্য অ্যাকাউন্ট করে, তাই আপনি এটিকে উপেক্ষা করতে পারেন। আপনার কেবলমাত্র পৃষ্ঠের অঞ্চল এবং তাপমাত্রার পার্থক্যের প্রভাবের জন্য অ্যাকাউন্ট করতে হবে।

আর এর এসআই ইউনিটগুলিKm2W

পৃষ্ঠের ক্ষেত্রফল বাড়ার সাথে সাথে তাপ স্থানান্তর বৃদ্ধি পায়, তাই Q হিসাবে গণনা করা হয়:

Qinsulation=A(TfTi)R
এবং
Qinsulation=(TfTi)R

উফফ, আমার মন্তব্য মুছতে পারে না তাই আমি একটি নতুন তৈরি করছি। আমি এটিকে বোঝাতে চাইছিলাম: নলাকার স্থানাঙ্কগুলিতে আমি আর-মানটি কীভাবে বাস্তবায়ন করব যদিও আপনি নিজের আর-মানগুলির প্রতিটি পৃষ্ঠের ক্ষেত্রটি পৃথকভাবে উল্লেখ করেছেন? যেমন আর = 1 / (এইচ ওয়াটার * 2 * পাই * আর 1 * এল) + রিিনুলেশন / (এখানে কী অঞ্চল) + 1 / (হাইআর * 2 * পাই * আর 2 * এল)। আমি কী আমার ইনসুলেশন আর-মানকে 2 * পাই * এল দিয়ে ভাগ করব যেখানে এলটি ট্যাঙ্কের উচ্চতা?
মিচই

আপনি কি ট্যাঙ্কের অভ্যন্তরে তাপমাত্রার প্রকরণটি মডেল করার চেষ্টা করছেন, বা মোট তাপ স্থানান্তর সন্ধান করার জন্য? সাধারণত আপনি কেবল আর মানগুলি যোগ করতে চান তবে নলাকার স্থানাঙ্কের ক্ষেত্রে, আমি মনে করি না যে অঞ্চলটি স্থির নয় বলে আপনি করতে পারবেন। দুঃখিত আমি সেখানে খুব একটা সাহায্য করি না।
অদ্ভুতভাবে

আমি সিলিন্ড্রিকাল স্থানাঙ্কে ট্যাঙ্কের মাধ্যমে মোট তাপ স্থানান্তর বের করার চেষ্টা করছি। আমার কাছে একটি মডেল রয়েছে তবে আমি এখনই তাপমাত্রার প্রোফাইলে বিশেষ আগ্রহী নই। এটি অবশ্যই কোথাও করা হয়েছে যেহেতু উত্তপ্ত পানির ট্যাঙ্কগুলিতে অন্তরণ জন্য আর-মানগুলি সাধারণ। তারা কীভাবে তাদের আর-মানটি ব্যবহার করে?
মিচই

এটি যদি আমি থাকতাম তবে আমি পানির অভ্যন্তরে একটি ধ্রুবক তাপমাত্রা গ্রহণ করতাম - থার্মোস্টেটটি যা সেট করা হোক না কেন। তাপমাত্রার গ্রেডিয়েন্ট সম্ভবত গণনায় খুব বেশি পার্থক্য আনবে না, যদিও আপনি কী ধরণের নির্ভুলতার সন্ধান করছেন তা আমি নিশ্চিত নই। তারপরে আপনি অন্য কোথাও পাওয়া সামগ্রিক পরিবাহনের সাথে অন্তরণ পরিবাহের সংক্ষিপ্তসার করে আমি মোট আর মান তৈরি করব। এটা বেশ সোজা মনে হচ্ছে।
অদ্ভুতভাবে

: আমি এই ব্যাখ্যা আমি খুঁজছেন ছিল মনে physics.stackexchange.com/questions/348770/...
MechE

1

@ জিকলি: সম্মত এছাড়াও, নিরোধক বেধটি বাহ্যিক সংক্রমণের জন্য পৃষ্ঠের ক্ষেত্রফলকে বাড়িয়ে দেবে, যেমন: এই প্রভাবটি ঘূর্ণিঝড়গুলির জন্য অন্তরণগুলির সমালোচনামূলক ব্যাসার্ধকে উত্সাহিত করেছিল: https://en.wikedia.org/wiki/Thermal_insulation :

Qjacketair=2π(lheater)(rheater+tjacket)h(TjacketTair)
rcritical=kh=tjacketRjacketh
আমাদের সাইট ব্যবহার করে, আপনি স্বীকার করেছেন যে আপনি আমাদের কুকি নীতি এবং গোপনীয়তা নীতিটি পড়েছেন এবং বুঝতে পেরেছেন ।
Licensed under cc by-sa 3.0 with attribution required.